Difference between revisions of "2020 AMC 12A Problems/Problem 15"

m (Solution 1)
m (Solution 1)
 
Line 74: Line 74:
 
draw((xMin,0)--(xMax,0),black+linewidth(1.5),EndArrow(5));
 
draw((xMin,0)--(xMax,0),black+linewidth(1.5),EndArrow(5));
 
draw((0,yMin)--(0,yMax),black+linewidth(1.5),EndArrow(5));
 
draw((0,yMin)--(0,yMax),black+linewidth(1.5),EndArrow(5));
label("Re",(xMax,0),E);
+
label("Re",(xMax,0),(2,0));
label("Im",(0,yMax),N);
+
label("Im",(0,yMax),(0,2));
  
 
//The n such that we're taking the nth roots of unity multiplied by 2.
 
//The n such that we're taking the nth roots of unity multiplied by 2.

Latest revision as of 12:38, 13 September 2021

Problem

In the complex plane, let $A$ be the set of solutions to $z^{3}-8=0$ and let $B$ be the set of solutions to $z^{3}-8z^{2}-8z+64=0.$ What is the greatest distance between a point of $A$ and a point of $B?$

$\textbf{(A) } 2\sqrt{3} \qquad \textbf{(B) } 6 \qquad \textbf{(C) } 9 \qquad \textbf{(D) } 2\sqrt{21} \qquad \textbf{(E) } 9+\sqrt{3}$

Solution 1

We solve each equation separately:

  1. We solve $z^{3}-8=0$ by De Moivre's Theorem.

    Let $z=r(\cos\theta+i\sin\theta)=r\operatorname{cis}\theta,$ where $r$ is the magnitude of $z$ such that $r\geq0,$ and $\theta$ is the argument of $z$ such that $0\leq\theta<2\pi.$

    We have \[z^3=r^3\operatorname{cis}(3\theta)=8(1),\] from which

    • $r^3=8,$ so $r=2.$
    • $\begin{cases} \begin{aligned} \cos(3\theta) &= 1 \\ \sin(3\theta) &= 0 \end{aligned}, \end{cases}$ so $3\theta=0,2\pi,4\pi,$ or $\theta=0,\frac{2\pi}{3},\frac{4\pi}{3}.$
    The set of solutions to $z^{3}-8=0$ is $\boldsymbol{A=\left\{2,-1+\sqrt{3}i,-1-\sqrt{3}i\right\}}.$ In the complex plane, the solutions form the vertices of an equilateral triangle whose circumcircle has center $0$ and radius $2.$
  2. We solve $z^{3}-8z^{2}-8z+64=0$ by factoring by grouping.
  3. We have \begin{align*} z^2(z-8)-8(z-8)&=0 \\ \bigl(z^2-8\bigr)(z-8)&=0. \end{align*} The set of solutions to $z^{3}-8z^{2}-8z+64=0$ is $\boldsymbol{B=\left\{2\sqrt{2},-2\sqrt{2},8\right\}}.$

In the graph below, the points in set $A$ are shown in red, and the points in set $B$ are shown in blue. The greatest distance between a point of $A$ and a point of $B$ is the distance between $-1\pm\sqrt{3}i$ to $8,$ as shown in the dashed line segments. [asy] /* Made by MRENTHUSIASM */ size(200);   int xMin = -10; int xMax = 10; int yMin = -10; int yMax = 10; int numRays = 24;  //Draws a polar grid that goes out to a number of circles  //equal to big, with numRays specifying the number of rays:  void polarGrid(int big, int numRays)  {   for (int i = 1; i < big+1; ++i)   {     draw(Circle((0,0),i), gray+linewidth(0.4));   }   for(int i=0;i<numRays;++i)    draw(rotate(i*360/numRays)*((-big,0)--(big,0)), gray+linewidth(0.4)); }  //Draws the horizontal gridlines void horizontalLines() {   for (int i = yMin+1; i < yMax; ++i)   {     draw((xMin,i)--(xMax,i), mediumgray+linewidth(0.4));   } }  //Draws the vertical gridlines void verticalLines() {   for (int i = xMin+1; i < xMax; ++i)   {     draw((i,yMin)--(i,yMax), mediumgray+linewidth(0.4));   } }  horizontalLines(); verticalLines(); polarGrid(xMax,numRays); draw((xMin,0)--(xMax,0),black+linewidth(1.5),EndArrow(5)); draw((0,yMin)--(0,yMax),black+linewidth(1.5),EndArrow(5)); label("Re",(xMax,0),(2,0)); label("Im",(0,yMax),(0,2));  //The n such that we're taking the nth roots of unity multiplied by 2. int n = 3;  pair A[]; for(int i = 0; i <= n-1; i+=1) {   A[i] = rotate(360*i/n)*(2,0); }  draw(Circle((0,0),2),red); draw(A[1]--(8,0),dashed); draw(A[2]--(8,0),dashed);  for(int i = 0; i< n; ++i) dot(A[i],red+linewidth(4.5));   dot((2*sqrt(2),0),blue+linewidth(4.5)); dot((-2*sqrt(2),0),blue+linewidth(4.5)); dot((8,0),blue+linewidth(4.5)); [/asy] By the Distance Formula, the answer is \[\sqrt{(-1-8)^2+\left(\pm\sqrt{3}-0\right)^2}=\sqrt{84}=\boxed{\textbf{(D) } 2\sqrt{21}}.\] ~lopkiloinm ~MRENTHUSIASM

Solution 2

Alternatively, we can solve $z^{3}-8=0$ by the difference of cubes: \[(z-2)\left(z^2+2z+4\right)=0.\]

  • If $z-2=0,$ then $z=2.$
  • If $z^2+2z+4=0,$ then $z=-1\pm\sqrt{3}i$ by either completing the square or the quadratic formula.

The set of solutions to $z^{3}-8=0$ is $A=\left\{2,-1+\sqrt{3}i,-1-\sqrt{3}i\right\}.$

Following the rest of Solution 1 gives the answer $\boxed{\textbf{(D) } 2\sqrt{21}}.$

~MRENTHUSIASM

See Also

2020 AMC 12A (ProblemsAnswer KeyResources)
Preceded by
Problem 14
Followed by
Problem 16
1 2 3 4 5 6 7 8 9 10 11 12 13 14 15 16 17 18 19 20 21 22 23 24 25
All AMC 12 Problems and Solutions

The problems on this page are copyrighted by the Mathematical Association of America's American Mathematics Competitions. AMC logo.png